disuguaglianza, algebra (?)

Polinomi, disuguaglianze, numeri complessi, ...
Rispondi
Avatar utente
mod_2
Messaggi: 726
Iscritto il: 18 ago 2007, 20:26
Località: In fondo a destra

disuguaglianza, algebra (?)

Messaggio da mod_2 »

Dati n numeri positivi $ $a_1,~a_2,~...~a_n $ ed un intero non negativo k, dimostrare che
$ $\frac{a_1^k+a_2^k+...+a_n^k}{n}\leq\frac{a_1^{k+1}+a_2^{k+1}+...+a_n^{k+1}}{a_1+a_2+...+a_n} $
Appassionatamente BTA 197!
¬[ƒ(Gabriel)³²¹º]¼+½=¾
Messaggi: 849
Iscritto il: 22 ott 2006, 14:36
Località: Carrara/Pisa

Messaggio da ¬[ƒ(Gabriel)³²¹º]¼+½=¾ »

dividendo LHS per la media k-esima elevata alla k+1 e RHS per la media k+1-esima elevata alla k+1 rimane media k-esima maggiore o uguale a AM che è vera.
Alex89
Messaggi: 366
Iscritto il: 29 gen 2006, 16:57

Messaggio da Alex89 »

Oppure:

$ $\frac{a_1^k+a_2^k+...+a_n^k}{n}\leq\frac{a_1^{k+1}+a_2^{k+1}+...+a_n^{k+1}}{a_1+a_2+...+a_n}* \frac{n}{n} $

$ \displaystyle \frac{a_1^k+a_2^k+...+a_n^k}{n}*\frac{a_1+a_2+...+a_n}{n}\leq \frac{a_1^{k+1}+a_2^{k+1}+...+a_n^{k+1}}{n} $

ossia
$ [Media (K)]^k(AM) \leq [Media (K+1)]^k [Media (K+1)] $

e questa è la disuguaglianza delle medie p-esime.
Avatar utente
mod_2
Messaggi: 726
Iscritto il: 18 ago 2007, 20:26
Località: In fondo a destra

Messaggio da mod_2 »

ok, è troppo semplice.:D
Rilancio: Dimostrarlo nel maggior numero di modi possibili.
Appassionatamente BTA 197!
Avatar utente
exodd
Messaggi: 728
Iscritto il: 09 mar 2007, 19:46
Località: sulle pendici della provincia più alta d'europa

Messaggio da exodd »

aggiungo:
trovare una soluzione senza usare medie k-esime
Tutto è possibile: L'impossibile richiede solo più tempo
julio14 ha scritto: jordan è in realtà l'origine e il fine di tutti i mali in $ \mathbb{N} $
EvaristeG ha scritto:Quindi la logica non ci capisce un'allegra e convergente mazza.
ispiratore del BTA

in geometry, angles are angels

"la traslazione non è altro che un'omotetia di centro infinito e k... molto strano"
g(n)
Messaggi: 109
Iscritto il: 14 ott 2007, 19:24
Località: Codroipo, il paese più anagrammato d'Italia

Messaggio da g(n) »

Moltiplicando per i denominatori si arriva a
$ \displaystyle \sum a_i \sum a_i^{k}\leq n \sum a_i^{k+1} $
che è vera per Chebycheff considerato che le n-uple $ (a_1,..,a_n) $ e $ (a_1^k,...,a_n^k) $sono ordinate allo stesso modo
Avatar utente
Anér
Messaggi: 722
Iscritto il: 03 giu 2008, 21:16
Località: Sabaudia

Messaggio da Anér »

Innanzitutto mi presento: sono Andrea Bianchi, e benché mi sia iscritto a giugno sul forum, questa è la prima volta che aggiungo un post (WOWWW!!!).
E se per il problema di cui sopra non conoscessimo né le diseguaglianze tra le medie, né la disuguaglianza di Chebycheff, né qualsiasi altra diseguaglianza famosa? Qualcuno provi a fare la dimostrazione con la trigonometria!
Avatar utente
Anér
Messaggi: 722
Iscritto il: 03 giu 2008, 21:16
Località: Sabaudia

Messaggio da Anér »

Intendevo rinnovare l'invito a trovare altre dimostrazioni, e sarebbe più che sorprendente se qualcuno usasse la trigonometria. Non intendo dire che io sarei in grado di dare la dimostrazione con la trigonometria!
Barsanti
Moderatore
Messaggi: 263
Iscritto il: 01 gen 1970, 01:00
Località: Pisa

Messaggio da Barsanti »

magari l'induzione?
Avatar utente
Anér
Messaggi: 722
Iscritto il: 03 giu 2008, 21:16
Località: Sabaudia

Messaggio da Anér »

Oh, che bella idea! E dire che l'induzione si può fare sia su n che su k!
Sono il cuoco della nazionale!
Rispondi